Practice Exam Questions Flashcards

1
Q

When must an insurable interest exist for a life insurance policy?

A) Both at the time of death and at the inception of the policy
B) At the time the beneficiary is paid
C) At the inception of the policy
D) At the time of death

A

The correct answer is (C).

An insurable interest for a life insurance policy must exist at the inception of the policy but not necessarily at the time of loss.

How well did you know this?
1
Not at all
2
3
4
5
Perfectly
2
Q

Tara wants to fully understand insurance contracts before she buys one. She asks for an explanation of the legal characteristics of an insurance contract. All of the following statements are correct EXCEPT:

A) An insurance contract is a contract of adhesion, that is, the insured accepts the contract as written.

B)An insurance contract is aleatory in the sense that equal monetary values are exchanged.

C) An insurance contract is unilateral, that is, only one party agrees to a legally enforceable promise.

D) An insurance contract is conditional, that is, an insured must meet all conditions of the policy in order to claim payment.

A

The correct answer is (B).

All of the statements are correct regarding the characteristics of a contract except option (B). An aleatory contract has values exchanged that are unequal (small premium versus potential large claim).

How well did you know this?
1
Not at all
2
3
4
5
Perfectly
3
Q

Amber owns a house with a replacement cost of $500,000. She purchases $300,000 of insurance with an 80 percent coinsurance requirement and a $500 deductible. If Amber’s house is hit by a hurricane and she suffers a $150,000 loss, what will the insurer pay?

A) $74,500

B) $112,000

C) $119,500

D) $149,500

A

The correct answer is (B).

(amount of coverage purchased ÷ coinsurance) × loss

[$300,000 ÷ ($500,000 × 0.80)] × $150,000 = $112,500 − $500 deductible = $112,000

How well did you know this?
1
Not at all
2
3
4
5
Perfectly
4
Q

Shira has a major medical policy with a $500 annual deductible, a $2,000 out-of-pocket limit on covered losses excluding the deductible, and an 80/20 coinsurance provision. Shira breaks a leg dancing and has surgery that costs $12,000. How much will she need to pay for the surgery?

A)$500

B) $2,300

C) $2,500

D) $2,800

A

The correct answer is (C).

Shira is responsible for a $500 annual deductible and 20 percent of the costs. However, she has an out-of-pocket limit of $2,000 excluding the deductible.

$12,000 − $500 = $11,500

$11,500 × 0.20 = $2,300

$2,000 + $500 = $2,500

How well did you know this?
1
Not at all
2
3
4
5
Perfectly
5
Q

Regarding the advantages of a preferred-provider option (PPO), all of the following are correct EXCEPT:

A) A gatekeeper is required for specialist consultations.

B) A primary care physician is not required.

C) Health care costs are low when the insured uses in-network providers.

D) Yearly out-of-pocket costs are limited.

A

The answer is (A).

An advantage of a PPO is that there is no gatekeeper or primary care physician that serves in that role.

How well did you know this?
1
Not at all
2
3
4
5
Perfectly
6
Q

All of the following are basic characteristics of health maintenance organizations (HMOs) EXCEPT

A) the use of an organized system to deliver medical services to the members.

B) broad, comprehensive health services that are provided to the members.

C) an emphasis on cost containment through the use of coinsurance and large deductibles.

D) a specified geographic region in which members must live.

A

The answer is (C).

Large deductibles and coinsurance are generally not emphasized in HMOs.

How well did you know this?
1
Not at all
2
3
4
5
Perfectly
7
Q

Henry is covered under his employer’s major medical expense plan. The plan has a calendar-year deductible of $1,000, a 75 percent coinsurance provision that applies to the next $8,000 of covered expenses, and full coverage for any remaining covered expenses. If Henry incurs covered medical expenses of $15,000 during the year, how much will be paid by his employer’s plan?

A) $10,500

B) $12,000

C) $13,000

D) $14,000

A

The answer is (B).

His employer’s plan will pay $12,000. If Henry incurs $15,000 of covered medical expenses, he must pay $1,000 out-of-pocket to meet the deductible. Of the remaining $14,000 of expenses, $8,000 of it is subject to the 75 percent coinsurance provision. This means that Henry must pay another $2,000 (that is, .25 × $8,000 = $2,000) out-of-pocket for a total amount of $3,000 (that is, $1,000 deductible + $2,000 coinsurance = $3,000). Subtracting $3,000 from $15,000 leaves $12,000 for the medical expense plan to cover.

How well did you know this?
1
Not at all
2
3
4
5
Perfectly
8
Q

Hanna, aged 59½, was employed full-time at AWY Corporation, but she was laid off during a recent downsizing. The company has 25 full-time employees and 10 part-time employees, but it does not offer group health insurance. What should Hanna do for health insurance while unemployed?

A) Elect 18 months of COBRA coverage.

B)Elect COBRA for up to 36 months or until she has group coverage with a new employer.

C) Elect Medicare coverage.

D) Purchase an individual major medical policy until she can obtain group coverage with a new employer.

A

The correct answer is (D).

The employer is not required to offer COBRA because it does not have group health insurance. Hanna is not eligible for Medicare until age 65.

How well did you know this?
1
Not at all
2
3
4
5
Perfectly
9
Q

All of the following statements regarding flexible spending accounts (FSAs) are correct EXCEPT: 

A) Funds may be used for optional medical procedures like LASIK eye surgery. 

B) Funds may be used for child care.

C) Funds must be used by the end of the calendar year.

D) The accounts are funded with pretax dollars. 

A

The answer is (C).

Money in FSA accounts must be used by 2 ½ months after the end of the plan year or the funds are lost.

How well did you know this?
1
Not at all
2
3
4
5
Perfectly
10
Q

Members of a three-person partnership want to enter into a buy-sell arrangement. How many life insurance policies would need to be purchased to properly fund coverage of the partnership using a cross-purchase agreement?

A) One policy

B) Three policies

C) Six policies

D) Nine policies

A

The correct answer is (C).

Each owner would need to purchase a life insurance policy on the other owners, and therefore a total of six policies would need to be purchased.

3 × (3 − 1) = 3 × 2 = 6

How well did you know this?
1
Not at all
2
3
4
5
Perfectly
11
Q

Which of the following life insurance policies contains a cash value savings component that reaches the face value of the policy at age 100 or 120?

A) A term policy

B) A whole life insurance policy

C) A universal life insurance policy

D) A lifetime annuity

A

The correct answer is (B).

Owners of whole life insurance policies pay premiums until age 100 or 120, when the cash value equals the face value of the policy.

How well did you know this?
1
Not at all
2
3
4
5
Perfectly
12
Q

Eugene has been advised by his insurance agent to purchase a variable universal life insurance policy. He has sought your advice regarding this purchase. All of the following are characteristics of a variable universal policy EXCEPT:

A) The policy features increasing or decreasing death benefits and flexibility of variable premium payments.

B) The policy owner has exclusive investment control over the cash value of the policy.

C) The death benefit is guaranteed to be equal to the face value.

D) The cash value of a variable universal life policy is dependent on premiums and investment returns.

A

The correct answer is (C).

All statements are true except option (C). The minimum death benefit of the variable universal life policy is guaranteed, but the death benefit can increase if the investment experience on cash value is good.

How well did you know this?
1
Not at all
2
3
4
5
Perfectly
13
Q

All of the following statements regarding a MEC are correct EXCEPT:

A) A life insurance policy that fails the seven-pay test is deemed a MEC.

B) Withdrawals from a MEC are subject to FIFO tax treatment.

C) A 10 percent penalty applies to withdrawals taken from a MEC prior to the owner reaching age 59½.

D) The primary issue with a MEC is the taxation of withdrawals (loans) because the death benefit is generally tax-free.

A

The correct answer is (B).

MECs are subject to LIFO tax treatment.

How well did you know this?
1
Not at all
2
3
4
5
Perfectly
14
Q

Which of the following statements concerning the operation of a life insurance policy is correct?

A) If the insured dies during the grace period, the insurer refunds only the premiums paid.

B) If after the insured’s death it is discovered that the insured’s age had been misstated, the insurer will pay the beneficiary the amount that the premiums paid would have purchased at the insured’s correct age.

C) Generally, a policy can be reinstated after it has been surrendered for its cash value as long as evidence of insurability is provided to the insurer.

D) Once a life insurance policy has been issued, it is incontestable on the basis of material misrepresentation or concealment.

A

The correct answer is (B).

If after the insured’s death it is discovered that the insured’s age had been misstated, the insurer will pay the beneficiary the amount that the premiums paid would have purchased at the insured’s correct age. Option (A) is incorrect because if the insured dies during the grace period, full policy benefits are paid, but the insurer may deduct the overdue premium. Option (C) is incorrect because reinstatement applies to policies that have lapsed. Option (D) is incorrect because the insurer can contest the validity of a policy during the contestable period.

How well did you know this?
1
Not at all
2
3
4
5
Perfectly
15
Q

All of the following statements concerning disability income insurance are correct EXCEPT:

A) Premiums for disability income insurance coverage are a function of the insured’s health, biological sex, age, and the level of income benefits provided by the policy.

B) To qualify for disability income, one must become totally disabled while the policy is in force and remain so until the elimination (exclusion) period has ended.

C) A policy that integrates with Social Security will reduce payable benefits by the amount of Social Security the person with a disability is eligible to receive.

D) The Social Security program requires the person who is disabled to wait at least one month before receiving benefits.

A

The correct answer is (D).

In order to receive disability benefits, a person must be disabled for 5 months, must be unable to perform the duties of any occupation, and must have a disability that is either expected to last for 12 months or to result in death.

How well did you know this?
1
Not at all
2
3
4
5
Perfectly
16
Q

Nia, a technician, buys a disability policy with a base benefit of $6,000 and an SIS offset benefit of $1,200. Nia becomes disabled and eventually receives $1,000 in Social Security disability benefits. How much will she receive from the insurance company after Social Security benefits begin?

A) $4,800

B) $6,000

C) $6,200

D) $7,200

A

The correct answer is (C).

Initially, the policy will pay the sum of the base benefit and the SIS benefit, which equals $7,200. After Social Security begins paying, the insurance company will only pay $6,200. The total benefit is offset by the amount received from the SSA.

How well did you know this?
1
Not at all
2
3
4
5
Perfectly
17
Q

All of the following regarding short-term disability coverage and long-term disability coverage are true EXCEPT: 

A) It is not important to coordinate short-term disability coverage with long-term disability coverage. 

B) Short-term coverage may range from 1 to 2 years. 

C) Long-term benefits may continue until retirement or death. 

D) The combination of short- and long-term coverages are common in group insurance offerings.

A

The correct answer is (A).

The benefits of short-term disability coverage and the benefits of long-term disability coverage should be coordinated so that there is no gap in coverage between short-term disability and long-term disability.

How well did you know this?
1
Not at all
2
3
4
5
Perfectly
18
Q

Kim earns $100,000 per year as a NASA engineer. He has a loss-of-income disability policy with a benefit of $5,000 per month. He suffers a disability to his right leg while at work. As a result of his injury, he needs to work at a desk job, earning $60,000. How much in disability benefits will he receive?

A) $5,000 per month

B) $40,000 per year

C) $2,000 per month

D) $0 since he is less than 50 percent disabled

A

The correct answer is (C).

Kim is partially disabled, and he lost 40 percent of his income. Therefore, he will receive a benefit equal to 40 percent of his monthly disability policy benefit, or $2,000 per month.

How well did you know this?
1
Not at all
2
3
4
5
Perfectly
19
Q

Carlotta is struck in the rear of her candy-apple-red sports car. Her injuries are severe, leaving her unable to work as a hostess at a high-end restaurant in New Orleans. The restaurant offers employees disability insurance, which Carlotta has elected as part of her employee benefits package. She pays for 50 percent of the insurance on a pretax basis. The employer pays the remainder of the policy premium. Her monthly benefit is $2,500 per month. Which of the following is correct?

A) One hundred percent of her benefit will be subject to income tax.

B) Two-thirds of her benefit will be subject to income tax.

C) Fifty percent of her benefit will be subject to income tax.

D) None of her benefit will be subject to income tax.

A

The correct answer is (A).

All of her premiums are paid on a pretax basis , therefore 100 percent of her benefit will be subject to income tax.

How well did you know this?
1
Not at all
2
3
4
5
Perfectly
20
Q

Long-term care needs are important for those who are aged 65 and older. Which of the following statements is correct regarding people who are 65 years old or older?

A) Fifty percent of people over age 65 will need long-term care at some point in their lives.

B) Men are more likely than women to be ADL-disabled.

C) It is more likely that a woman will be ADL disabled than have an auto accident.

D) Ninety percent of people over age 65 will need long-term care at some point in their lives.

A

The correct answer is (C).

LTC needs are much more likely than auto accidents or house fires, and women are more likely than men to need help with activities of daily living. Seventy percent of people over the age of 65 will need long-term care at some point in their lives.

21
Q

Which of the following statements concerning the need for long-term care insurance is (are) correct?

I. Private medical expense insurance policies (both group and individual), in some cases, provide coverage only if a person also needs medical care; however, benefits are not provided if a person is merely a senior citizen “old” and needs someone to care for him or her.

II. Medicare is inadequate to cover long-term care because it does not cover custodial care if that is all that is needed.

A) I only

B)II only

C) Both I and II

D) Neither I nor II

A

The correct answer is (C).

Neither private medical expense insurance nor Medicare is adequate to cover the risks of long-term care.

22
Q

Today, many long-term care policies are treated as tax-qualified contracts. All of the following are true regarding tax-qualified long-term care contracts EXCEPT:

A) Tax-qualified long-term care policies must provide benefits that are limited to long-term care services.

B) These policies can be provided under an employer-sponsored cafeteria plan.

C) These policies allow employers to provide this benefit and take a current income tax deduction, and the policies allow the employee to avoid income inclusion.

D) The premiums for these policies may be deductible either above-the-line or below-the-line.

A

The correct answer is (B).

These policies cannot be included in a cafeteria plan. All other statements are correct.

23
Q

Sophia receives some long-term care services from her daughter, Dorothy, at their Florida home. Sophia has a long-term care policy with a maximum $200/day benefit. How much can Dorothy receive as payment from this policy if she is not a licensed long-term care provider? 

A) $0

B) $100 

C) $200 

D) Up to $200/day

A

The correct answer is (A).

Long-term care policies do not pay for care provided by family members.

24
Q

Therese, aged 75, is a widow with no close relatives. She is very ill, unable to walk, and confined to a custodial nursing home. Which of the following statements is true with regard to which program(s) is (are) likely to pay benefits toward the cost of the nursing home?

I. Medicare may pay for up to 80 additional days of care after a 20-day deductible.

II. Medicaid may pay if the client has income and assets below state-mandated thresholds.

A) I only

B) II only

C) Both I and II

D) Neither I nor II

A

The correct answer is (B).

Statement I is incorrect because Medicare covers all costs for the first 20 days of skilled nursing home care and covers the next 80 days with a deductible.

25
Q

What is the goal of long-term care?

A) Return the recipient to independent living.

B) Cure multiple chronic conditions.

C) Promote functional independence.

D) Make a terminal person comfortable.

A

The correct answer is (C).

Long-term care is used to help promote functional independence.

26
Q

Annuities are complex. Which of the following statements is correct regarding annuities?

A) All annuities must be annuitized or exchanged.

B) Annuities that are not annuitized do not mitigate the risk of superannuation.

C) One of the advantages of investing in annuities is that they are not subject to required minimum distribution rules.

D) Annuities are included in the gross estate of the decedent at the death of the owner.

A

B is correct

Annuities that are not annuitized do not mitigate the risk of superannuation. Option (A) is incorrect because annuities could be surrendered, not just exchanged or annuitized. Choice (C) is incorrect as qualified annuities are subject to minimum distribution rules. Choice (D) is incorrect as annuities that cease making payments at death are not included in the decedent’s gross estate.

27
Q

Taylor and Avery Blake wish to purchase an annuity that provides income as long as at least one of them is alive. They have no children and no interest in passing their assets to heirs. Which of the following annuities is best suited for their purposes?

A) A cash -refund annuity

B) An installment-refund annuity

C) A joint-and-last-survivor annuity

D) A joint-life annuity

A

The answer is (C).

A joint-and-last-survivor annuity is best suited to the Blakes’ purposes. Option (A) is incorrect because providing a lump-sum cash refund to heirs is inconsistent with the Blakes’ goals. Option (B) is incorrect because providing an installment refund to heirs is inconsistent with their goals. Option (D) is incorrect because joint-life annuity payments would cease when either Taylor or Avery dies, rather than providing an income for as long as at least one of them is alive.

28
Q

Cade purchased a life insurance policy to ensure that his children were provided for in the event of his untimely death. Since they are now grown, he feels he no longer needs the life insurance. He would like to exchange it for an annuity that can provide him additional income for his retirement years. Which of the following is correct?

A) He can make the exchange, but it will be taxable to the extent of the cash value.

B) He can make the exchange, but it will be taxable to the extent of the cash value less the additional money he puts into the annuity.

C) He can make the exchange, which will not be taxable.

D) He can make the exchange, which will not be taxable, but his basis will not reflect any of the investment into the life insurance policy.

A

The correct answer is (C).

An exchange of a life policy to an annuity is a tax-free exchange under Section 1035. The basis will include the original investment and the additional funds.

29
Q

Scott purchased a single-premium annuity for $200,000. The annuity pays him $1,000 per month. If his life expectancy was 20 years when he purchased it, how much of each payment is NOT subject to tax?

A) $177

B) $475

C) $833

D) $1,000

A

The correct answer is (C).

The exclusion ratio is the owner’s investment in the contract divided by the expected return.

exclusion ratio = $200,000 ÷ (20 years × 12 months × $1,000) = 83.33%

taxable portion = 1− exclusion ratio = 16.67%

non-taxable portion of the payment = $833

30
Q

Monica is a 70-year-old widow with no dependents. She wants to invest in an annuity that will produce income now. She has $100,000 to invest and wants to receive the most she can in monthly income. Which of the following is the most suitable annuity for Monica based on her objectives?

A) A longevity annuity

B) A 20-year term-certain, fixed annuity

C) An immediate, single-premium life annuity

D) A deferred, fixed annuity

A

The correct answer is (C).

Monica wants the greatest amount of income now. A single-life annuity with no guarantee will provide her with the highest monthly income. A longevity annuity is usually deferred and any guarantees or use of a joint-life expectancy will reduce the monthly benefit.

31
Q

Darnell purchased a straight life annuity from Assurance Annuities using funds from his 401(k) plan. Which of the following is correct?

A) His annuity will make payments for his life and that of his beneficiary.

B) His annuity cannot be a variable annuity.

C) His annuity will be included in his gross estate at death.

D) His annuity will have no return of basis.

A

The correct answer is (D).

Option (D) is correct as pre-tax funds were used to purchase the annuity. Therefore, all the annuity payments will be taxable. Option (A) is incorrect as a straight-life annuity will pay for only one life. Option (B) is incorrect as it could be a variable annuity. Option (C) is incorrect as there will be no payments to include in his gross estate since the annuity ends at his death.

32
Q

Which of the following statements is correct with regard to longevity insurance?

I. It is a deferred annuity purchased at or near retirement that is usually payable at age 85. 

II. It is expensive due to the funding requirements at an advanced age. 

A) I only 

B) II only 

C) Both I and II 

D) Neither I nor II 

A

The correct answer is (A).

Usually this sophisticated arrangement involves the purchase of a deferred annuity at or near retirement with its distributions payable at age 85. This provides a client some level of comfort by assuring her that she will most likely not run out of money if she lives a prolonged period beyond life expectancy. Option II is incorrect because the cost is usually inexpensive for two reasons. First, the money has approximately 20 years or more to accumulate depending on the retirement age.  Second, the contracts do not usually start paying until age 85 and life expectancy is 81.  

33
Q

The exclusion ratio of an annuity

A) equals the owner’s investment in the annuity contract divided by the expected return on the annuity.

B) cannot be calculated for life annuities.

C) refers to the period of time until an annuity begins to pay benefits.

D) is particularly relevant for annuitants in low tax brackets.

A

The correct answer is (A).

The exclusion ratio equals the owner’s investment in the annuity contract divided by the expected return on the annuity. The resulting percentage is multiplied by the distribution, or payment, received to calculate the portion of the payment that is not subject to income tax. Option (B) is incorrect because the exclusion ratio can be calculated for all annuities. Option (D) is incorrect because the exclusion ratio helps determine how much of an annuity is subject to taxation; it is particularly relevant for annuitants in high tax brackets.

34
Q

All of the following are insured persons under the medical payments coverage of the personal auto policy (PAP) EXCEPT

A) the named insured’s friend who is injured while occupying the named insured’s covered auto.

B) a pedestrian who is struck and injured by the named insured’s covered auto.

C) the named insured who is injured while occupying a friend’s auto.

D) the named insured’s family member who is struck and injured by an auto while crossing the street.

A

The answer is (B).

Medical payments provide benefits for a covered vehicle’s occupants. It also covers the named insured or a family member who is a pedestrian struck by another vehicle. A pedestrian struck and injured by the named insured’s covered auto is covered under the insured’s liability coverage.

35
Q

Under an HO-3 policy, all open perils are covered, with some exceptions. All of the following are perils that are excluded from coverage under an HO-3 policy EXCEPT

A) termite damage.

B) flood damage.

C) earthquake damage.

D) tornado damage.

A

The correct answer is (D).

Tornado damage is covered. Termite damage is slow, so it is excluded. Flood damage and earthquake damage are definitely excluded.

36
Q

Which of the following individuals should consider purchasing an HO-8 policy?

A) Moira owns a classic Edwardian-style home, and the functional replacement cost of the home is less than the actual replacement cost.

B) Lukas owns a condominium in a resort community, and the functional replacement cost of the condo approximates its actual replacement cost.

C) Elijah owns a home in a development-style neighborhood, where many of the homes are similar to his.

D) Wyatt rents a home from his friend Lisa and would like coverage for his personal contents, loss of use, and potential liability.

A

The correct answer is (A).

HO-8 policies are appropriate when the functional replacement cost of the home is less than the actual replacement cost. Option (B) is incorrect because Lukas should have an HO-6 (Condominium) policy. Option (C) is incorrect because Elijah’s home can be easily replaced; the actual replacement cost should not exceed the functional replacement cost. Option (D) is incorrect because Wyatt should have an HO-4 (Renters) policy.

37
Q

All of the following are types of business and professional insurance EXCEPT

A) a commercial package policy (CPP).

B) a personal liability umbrella policy (PLUP). 

C) a business liability umbrella policy (BLUP).

D) a business owners policy (BOP).

A

The correct answer is (B).

While a PLUP offers many broad coverages, it does not cover professional or business activities. Specific professional and business coverages must be purchased.

38
Q

Noel, a resident of Louisiana, carries the minimum required PAP liability limit of 10/25/10. While on vacation in Massachusetts, Noel strikes a pedestrian with his car, resulting in medical costs of $45,000 for the victim. Massachusetts law requires motorists to maintain liability limits of 20/40/15. Which of the following statements concerning this situation is correct?

A) Noel’s insurance company will only pay $10,000 towards the victim’s medical costs.

B) Noel is subject to arrest or citation for failing to maintain proper insurance under Massachusetts law.

C) Noel may be personally liable for $25,000 of the medical expense costs.

D) Noel’s insurance company will pay $40,000 of the medial expense costs.

A

The correct answer is (C).

While driving in Massachusetts, Noel’s insurance will cover the minimum state requirements. Since the medical expense coverage per person is limited to $20,000 under Massachusetts law, Noel may be personally liable for the $25,000 difference between the covered amount and the total medical expenses. Options (A) and (B) are incorrect because the policy will pay the state-mandated minimum coverage. Option (D) is incorrect because $40,000 represents the maximum amount that the policy will pay for bodily injury per accident.

39
Q

An HO policy that covers all losses to covered property unless the loss is specifically excluded is

A) a basic named perils policy.

B) a broad named perils policy.

C) an open perils policy.

D) an HO-2 policy.

A

The correct answer is (C).

Open perils policies cover all losses except those that are specifically excluded. Named perils policies, whether basic or broad, cover perils that are specified in the contract. HO-2 policies cover both basic and broad named perils, but HO-2s are not open perils policies.

40
Q

A creditor makes a hard inquiry to a person’s credit report. For what length of time does the hard inquiry affect the FICO score?

A) 3 months

B) 6 months

C) 12 months

D) 24 months

A

The correct answer is (C).

The hard inquiry adversely affects the FICO score for 12 months.

41
Q

FICO scores are an indication of a person’s credit worthiness. Brian has a score of 500. How would you characterize his score?

A) Excellent

B) Good

C) Fair

D) Bad

A

The correct answer is (D).

The range for FICO scores is from 300 to 850. Anything below 550 is considered bad.

42
Q

Which type of inquiry may occur when renting an apartment or opening a checking or brokerage account?

I. A hard inquiry

II. A soft inquiry

A) I only

B) II only

C) Both I and II

D) Neither I nor II

A

The correct answer is (C).

Either type of inquiry could occur when renting an apartment or opening a checking or brokerage account.

43
Q

Which of the following concerning the Social Security system is correct?

A) Supplemental Security Income (SSI) benefits are funded by the U.S. Treasury, not Social Security taxes, as are the other benefits.

B) The Social Security retirement benefit is payable at full retirement age, with reduced benefits available as early as age 59 to anyone who has obtained at least a minimum amount of Social Security benefits.

C) The two Medicare trust funds are the federal Medical Insurance Trust Fund for Part A and the Supplementary Hospital Insurance Trust Fund for Part B of Medicare benefits.

D) Benefits can be paid to the dependent parents of a deceased insured worker when the parents are 59 years old or older.

A

The correct answer is (A).

The Social Security retirement benefit is payable at full retirement age with reduced benefits available as early as age 62 to anyone who has obtained at least a minimum amount of Social Security coverage. The two Medicare trust funds are the federal Hospital Insurance Trust Fund for Part A and the Supplementary Medical Insurance Trust Fund for Part B of Medicare benefits. Social Security benefits can be paid to the dependent parents of a deceased insured worker when the parents are 62 years old or older.

44
Q

John, aged 63, has enjoyed abundance during his lifetime and decided to share that abundance with three wives. His first marriage was to Karen Sandra, currently aged 65. This marriage lasted 25 years, and Sandra never remarried. Two years after divorcing Sandra, John married Barbara. who is currently aged 65. However, that marriage ended 3 years later when Barbara, divorced John and married another man. John and his current wife, Blanche, aged 58, have been married for 8 years. Which of the following statements concerning spousal benefits under Social Security is correct?

A) If John retires early this year, Blanche will be able to receive a spousal benefit equal to 50 percent of John’s Social Security benefit.

B) Barbara can collect a spousal benefit from John’s Social Security record.

C) Sandra can collect a spousal benefit from John’s Social Security record.

D) Blanche will be able to collect a spousal benefit on John’s record, as well as a Social Security benefit based on her own earnings when she reaches the age of 62.

A

The correct answer is (C).

Since John and Sandra’s marriage lasted more than 10 years, they have been divorced for at least 2 years, and Sandra has attained an appropriate age, she can collect a spousal benefit on John’s account. Option (A) is incorrect because Blanche is too young to collect a benefit this year. Option (B) is incorrect because John’s marriage to Barbara did not last 10 years. Option (D) is incorrect because Blanche will be able to collect the greater of the spousal benefit under John’s record or her PIA when she reaches Social Security retirement age.

45
Q

Quinton has been feeling unwell, and he, decides that his health would improve if he were to retire now. He is 68 years old, and his normal Social Security retirement age is 66. If he retires today and his normal age retirement PIA is $1,500, how much can Quinton expect to receive as a monthly retirement benefit?

A) $1,260

B) $1,500

C) $1,740

D) $1,750

A

The correct answer is (C).

Delayed benefits will increase at 8 percent per year of delay up until age 70. Therefore, Quinton’s benefit will be 16 percent higher than his PIA. The 8 percent increase is not compounded.

46
Q

Jules is single and received $28,000 of dividend income during the year. He also received $18,000 of Social Security benefits. Assuming this is the only income he received throughout the year, what portion of his Social Security benefits are taxable?

A) $0

B) $7,050

C) $9,000

D) $15,300

A

The correct answer is (B).

The taxable portion of Jules’s benefits is equal to the lesser of the following:

.85 × $18,000 = $15,300

or

0.85 × [$28,000 + (0.5 × 18,000) − $34,000] = $2,550 + $4,500 = $7,050

47
Q

All of the following statements concerning qualifying for Social Security retirement benefits are correct EXCEPT:

A) To qualify for retirement benefits, a worker must be fully insured.

B) Earning a designated amount of money, regardless of when it was earned during the year, will credit the worker with a quarter of coverage for that year.

C) A worker must have worked in six of the previous 13 quarters to qualify for retirement benefits.

D) A worker is fully insured if he earns 40 quarters of coverage.

A

The correct answer is (C).

For a worker’s family to qualify for survivors benefits, that worker must have worked in six of the previous 13 quarters. A worker must have earned 40 quarters of coverage to qualify for retirement benefits and be fully insured. A quarter of coverage, which is specified each year and is indexed, can be earned anytime during the year. The earnings must be subject to Social Security taxes.

48
Q

Which of the following statements concerning the reduction of Social Security benefits is (are) correct?

I. Besides early retirement, there are two other situations in which beneficiaries can have their benefits reduced: through the retirement earnings limitation test and through taxation of benefits.

II. Generally, a person can continue to work even though he or she is considered retired under Social Security; However, if the earnings of a person under normal retirement age exceed certain limitations, those earnings are subject to taxation.

A) I only

B) II only

C) Both I and II

D) Neither I nor II

A

The correct answer is (C).

Statement I is correct. The earnings limitation is about $18,240 (2020), although indexed each year. Recipients must give back $1 of benefits for each $2 that are in excess of the threshold. It only applies to those who are under normal age retirement at the time of the earnings. Statement II is correct for those under normal retirement age.